Uso de cantidades conservadas para violar el principio de incertidumbre

Si tengo un sistema de dos cuerpos (partículas A y B) del cual mido la cantidad de movimiento total, ¿puedo medir la posición de la partícula A con mucha precisión y la cantidad de movimiento de la partícula B con mucha precisión, y luego usar la cantidad de movimiento de la partícula B y el momento total (conservado) para darme el momento de la partícula A con incertidumbre satisfactoria Δ X Δ pag < / 2 , violando el principio de incertidumbre?

Se puede jugar un truco similar con cualquier cantidad conservada y su variable conjugada. ¿Qué paso o suposición es incorrecta? Creo que tal vez la diferencia entre medir el sistema completo y medir los subsistemas constituyentes no sea tan trivial como supuse.

Siguiendo el comentario de @AaronStevens, supongo que la pregunta puede reducirse a '¿Cambia la función de onda de una partícula si obtenemos nueva información sobre ella sin medirla directamente? ¿Si no, porque no?'.

Creo que aquí se ha hecho una pregunta similar ( Violación del principio de incertidumbre ) pero no se formuló correctamente. ¿Esto es quizás lo que el autor quiso decir con su pregunta?

El Δ los valores no son la precisión o exactitud en sus medidas aquí.
@AaronStevens ¿La implicación es que la función de onda de impulso de la partícula A no cambia con una medición en la partícula B? ¿Cómo puede ser ese el caso si tenemos nueva información sobre A?
No has obtenido nueva información. La cantidad de movimiento se conserva en promedio. Medir el impulso de una partícula no mide el impulso de la otra partícula. Estás poniendo algo de pensamiento clásico en tu pensamiento cuántico.
Posible duplicado de Violación del principio de incertidumbre La respuesta ahí es lo que quieres.
@AaronStevens Sí, vi la respuesta allí y también entiendo sus comentarios anteriores, pero me confunde que diga que no hemos obtenido nueva información. Seguramente, si se encuentra que la cantidad de movimiento de la partícula B tiene un pico brusco alrededor de un valor p dado y la cantidad de movimiento total tiene un pico bruscamente alrededor de P, entonces la función de onda de la partícula A será tal que las proyecciones sobre estados con momentos alrededor de (P - p ) serán grandes (o al menos más grandes de lo que habrían sido si no supiéramos nada acerca de B). No se conoce ninguna variable con precisión, por lo que no hay estados prohibidos, pero seguramente algunos son menos probables.
Si después de la medición inicial, las dos partículas tienen valores esperados de p igual a P/2 para un momento total P, y luego se encuentra que una partícula tiene un momento con un pico brusco alrededor de P, entonces si la función de onda de la primera partícula no cambia por este el nuevo impulso esperado para el sistema sería 3P/2. La medición repetida de este sistema arrojaría que, en promedio, el impulso no se conserva. eso es fisico?
Tienes que dar un ejemplo específico y detectores específicos. En los detectores que tenemos los errores de medida y momento son tales que la incertidumbre de Heisenberg (HUP) siempre se cumple, si pones los números. PERO no confunda las funciones de onda con las partículas. La función de onda Ψ Ψ da la probabilidad de encontrar una partícula en (x,y,z) . Una sola partícula medida da un punto que se suma al hacer el experimento muchas veces. El HUP funciona en los casos en que podemos medir y acumular distribuciones de probabilidad, confiamos en él a menos que el experimento lo invalide, en la actualidad nuestra precisión es mala
@anna_v Entendido pero, dados los principios de QM, HUP es una propiedad fundamental y derivable de las funciones de onda, no solo una observación experimental. Debería sostenerse incluso con detectores teóricamente perfectos.
Sí, pero no para un evento, para una acumulación de los mismos eventos de condición límite. Eso es lo que significan las distribuciones de probabilidad (que provienen de funciones de onda).

Respuestas (4)

Tome el ejemplo más simple posible, donde cada partícula tiene solo dos posiciones posibles y dos momentos posibles. Escribir METRO 1 y METRO 2 para los estados propios del operador de cantidad de movimiento. Escriba el estado del sistema como

i , j α i j METRO i METRO j

Entonces, si realiza una medición de momento en partículas A , obtendrás METRO 1 con probabilidad

pag = j α 1 j 2 / α i j 2

Por otro lado, primero podría hacer una medición de momento en partículas B y solo entonces realice una medición de cantidad de movimiento en la partícula A. La primera medición coloca al sistema en uno de los estados α i 1 METRO i METRO 1 o i α i 2 METRO i METRO 2 con probabilidades q 1 , q 2 proporcional a i α i 1 2 y i α i 2 2 . En cada uno de estos casos, puede calcular la probabilidad r i que su segunda medida produce METRO 1 . Así que la probabilidad de que su segunda medida produzca METRO 1 es q 1 r 1 + q 2 r 2 , que resulta (¡haz la aritmética!) ser exactamente lo mismo que pag .

En pocas palabras: medir la segunda partícula no puede brindarle ninguna información sobre la distribución de probabilidad de los resultados para una medición en la segunda.

Tu error: Asumir que cada partícula tenía un momento bien definido y una posición bien definida en primer lugar.

¿Puedo medir la posición de la partícula A con mucha precisión y la cantidad de movimiento de la partícula B con mucha precisión, y luego usar la cantidad de movimiento de la partícula B y la cantidad de movimiento total (conservada) para obtener la cantidad de movimiento de la partícula A?

Puede que esté leyendo esto incorrectamente, pero me parece que el problema con esta pregunta es la suposición que subyace a la sección en negrita.

No está claro exactamente lo que tiene en mente pero, independientemente de cómo se imagine cómo se realizan estas mediciones, parece haber una suposición de que el estado de A justo después de estas mediciones es un estado de (esencialmente) momento y posición definidos.

Pero no existe tal estado del que yo sea consciente. En particular, si A está en un estado de posición esencialmente definida, A simplemente no tiene un impulso bien definido para "darte".

Si tenía algo más en mente, aclare su pregunta y modificaré mi respuesta.

Suspiro, no vi la respuesta de WillO hasta después de publicarla. Ya que me tomé el tiempo de escribir, lo dejo aquí.
Creo que tienes razón, ahí es probablemente donde mi argumento se rompe (seguramente se rompe en alguna parte), pero no estoy muy seguro de por qué . Seguramente dada una medida exacta del impulso total y del impulso de B (que son ambas operaciones válidas dado que no nos importa la certeza de la posición), las mediciones repetidas en A darían consistentemente el valor esperado clásicamente. ¿Este último paso no es cierto? Si no es cierto, ¿por qué no viola la conservación de la cantidad de movimiento?
También parece factible que una vez que medimos la cantidad de movimiento total y la cantidad de movimiento de B podamos establecer la cantidad de movimiento de A; tenemos dos partículas y dos medidas. Sé que hay una diferencia entre los valores clásicos y los valores de expectativa cuántica, pero el punto sigue en pie.
Además, más respuestas siempre son mejores :)
@WuRuyi, si A está en una posición de estado de definición y B está en un estado de impulso definido, ¿es posible que el estado del sistema sea uno de impulso total definido?
Seguramente no, pero si el impulso se ha medido de antemano, ¿cómo puede haber cambiado? ¿La conservación del momento simplemente no es cierta para un solo conjunto de mediciones en la mecánica cuántica y, en cambio, solo es cierta en promedio después de muchas mediciones, algunas de las cuales tendrán un aumento del momento total y otras disminuirán? Por supuesto, este impulso total se refiere al valor esperado: si estamos en un estado propio de impulso de todo el sistema, probablemente no tenga sentido intentar realizar mediciones en las partes constituyentes debido a la incertidumbre de la posición...
@WuRuyi, en general, la sección de comentarios no es para una discusión extensa, pero sospecho que esto es lo que tienes en mente. ¿Quizás podría ampliar y extender su pregunta?

Hay dos casos. 1. las partículas están enredadas 2.las partículas no están enredadas.

Consideremos el primer caso. Por ejemplo, suponga que un electrón y un positrón chocan de tal manera que el impulso total antes de la colisión es cero, por lo que esperamos que después de la aniquilación , dos fotones creados se muevan de tal manera que el impulso total vuelva a ser cero, porque el impulso total se conserva. Eso significaría que si medimos el impulso de un fotón, ¡también adquiriríamos automáticamente el impulso del otro! Sin embargo, debido a que están entrelazados, si medimos uno, la función de onda del sistema colapsará y, por lo tanto, la posición de ambos se volverá completamente desconocida. Bueno, después de todo, en este tipo de sistemas no puede tener dos funciones de onda separadas, solo hay una función de onda para todo el sistema. Puedes decir lo mismo para la creación de pares.

En cuanto al segundo caso, no hay enredo. Las partículas tienen funciones de onda independientes. En este caso, no puede definir el momento total del sistema, a menos que mida el momento de cada partícula por separado. Y sabes, si haces eso, no puedes medir su posición después, porque se vuelve desconocida según el principio de Heisenberg. En este caso, sus impulsos no están relacionados y no hay forma de encontrar uno a partir del otro.

Si quieres, puedo entrar en detalles matemáticos. Pero la respuesta de @WillO cubre el segundo caso por completo.

Entonces, en el segundo caso, ¿está diciendo que no hay forma de medir el momento total del sistema con alta precisión sin medir cada uno individualmente?
@WuRuyi Si cree que hay una manera, ¡me gustaría escuchar eso! Podemos comprobar si es posible o no. Por supuesto, puede asignar un momento total probabilístico. Pero si haces eso, ¿cómo vas a usar la conservación del impulso?
No hay necesidad de dividir esto en dos casos. El Caso 2 es un caso especial del Caso 1.
@WillO, bueno, sí. En el segundo caso, puede dividir la función de onda de todo el sistema en dos funciones de onda para partículas, mientras que en el primero no es posible. Solo quería señalar que, de hecho, hay una manera de saber el momento de A del momento de B (en partículas entrelazadas), pero no viola el principio de Heisenberg.

Este fue un comentario, pero lo estoy promocionando a una segunda respuesta.

No hay absolutamente nada en esta pregunta que requiera dos partículas. Podría hacer exactamente la misma pregunta sobre una sola partícula en un estado propio de momento. Mide la posición de esa partícula, que podría ser cualquier cosa. Ahora que está en un estado propio de posición, mida su impulso, que podría ser cualquier cosa.

Este ejemplo tiene todas las características relevantes de su pregunta original, con las distracciones eliminadas.

Ahora tu pregunta es: "Si el impulso se ha medido de antemano, ¿cómo podría haber cambiado?". La respuesta es: podría haber cambiado si la mecánica cuántica es cierta.